nierówność z potęgami

Teoria liczb, teoria grafów, indukcja
Otrzymałeś(aś) rozwiązanie do zamieszczonego zadania? - podziękuj autorowi rozwiązania! Kliknij
maxkor
Czasem tu bywam
Czasem tu bywam
Posty: 126
Rejestracja: 07 cze 2015, 11:55
Podziękowania: 44 razy
Otrzymane podziękowania: 1 raz

nierówność z potęgami

Post autor: maxkor »

Wykaż ze dla \(n \in N, n>1\) mamy\( \left( \frac{1}{n} \right)^{n} + \left( \frac{2}{n} \right)^{n}+ \ldots+\left( \frac{n}{n} \right)^{n} <2\)
Awatar użytkownika
Jerry
Expert
Expert
Posty: 3525
Rejestracja: 18 maja 2009, 09:23
Podziękowania: 50 razy
Otrzymane podziękowania: 1930 razy

Re: nierówność z potęgami

Post autor: Jerry »

Dana nierówność jest równoważna
\(1^n+2^n+3^n+\ldots+(n-1)^n-n^n<0\)

i wg mnie dowód indukcyjny jest przyjazny... najistotniejszy moment:

\(L_T=1^{n+1}+2^{n+1}+3^{n+1}+\ldots+(n-1)^{n+1}+n^{n+1}-(n+1)^{n+1}=\\ \quad=
1^{n}+2\cdot 2^{n}+3\cdot3^{n}+\ldots+(n-1)\cdot(n-1)^{n}+n\cdot n^{n+1}-(n+1)^{n+1}<\\\quad <
[n\cdot1^{n}+n\cdot 2^{n}+n\cdot3^{n}+\ldots+n\cdot(n-1)^{n}]+n\cdot n^{n+1}-(n+1)^{n+1}\nad{z\, Z}{<}\\ \quad\nad{z\, Z}{<} n\cdot n^n +n^{n+1}-(n+1)^{n+1}=2n^{n+1}-(n+1)^{n+1}\nad{\color{red}{(*)}}{<}0=P_T\)

(*): Ze znanego faktu
\(\left(1+{1\over n}\right)^n\ge2\)
wynika
\(\left({n+1\over n}\right)^{n+1}\ge2(1+{1\over n})>2\\
(n+1)^{n+1}>2n^{n+1}\)

Pozdrawiam
ODPOWIEDZ